3
$\begingroup$

If $f: V \to W$ is a surjective homomorphism of vector spaces, and we have fixed a basis for $V$, it is always possible to find a basis for $W$ such that the matrix associated to $\phi$ in the two bases is triangular with ones on the "diagonal" (what I mean with this is explained more precisely later, in the case of abelian groups), up to possibly permuting the chosen basis of $V$. This motivates the following question, in the realm of finite abelian groups.

Let $H$ be a finite abelian group with a fixed basis $h_1, \ldots, h_n$, with "basis" here we mean elements that satisfy the property $\langle h_1 \rangle \oplus \ldots \oplus \langle h_n \rangle=H$. Let $\phi:H \to G$ be a homomorphism of finite abelian groups. Let us assume that $\phi$ is surjective, so $\phi(h_1), \ldots, \phi(h_n)$ generate $G$.

Is it possible to find a basis for $G$, and a permutation of the elements $h_1, \ldots, h_n$, such that the matrix associated with $\phi$ in the two given basis has a triangular form with $1$ on the diagonal?

More precisely, can we find a basis $g_1, \ldots, g_m$ for $G$, with $m \leq n$, and a permutation $\sigma \in \mathfrak{S}_n$ such that we can write

$\phi(h_{\sigma(i)}) = a_{i,1} g_1 + \ldots+ a_{i,m} g_m$

with $a_{i,j}=0$ when $j>i$, and $a_{i,i}=1$?

(I asked a similar question on math.stackexchange, but I spelled it differently and in a silly/wrong way so that the answer in that case was trivial)

$\endgroup$
5
  • 1
    $\begingroup$ Why don't you edit your question in math.stackexchange so that it says what you really mean? $\endgroup$ Sep 30, 2012 at 20:34
  • $\begingroup$ Here is my edited question in math.stackexchange. math.stackexchange.com/questions/192322/… $\endgroup$
    – calc
    Oct 1, 2012 at 7:26
  • $\begingroup$ What du you mean with a triangular matrix or a diagonal if V and W have different dimensions? $\endgroup$ Oct 3, 2012 at 11:36
  • 1
    $\begingroup$ I believe the answer to your question is yes, which should be possible to work out from the structure theorem for finite abelian groups. $\endgroup$ Oct 3, 2012 at 11:43
  • $\begingroup$ @Kofi I mean the same that I explain later in the case of abelian groups $\endgroup$
    – calc
    Oct 3, 2012 at 12:09

1 Answer 1

1
$\begingroup$

I think I can prove this in the case when $G$ is a $p$-group. Assume $\phi(h_1), \ldots, \phi(h_m)$ are irredundant generators: if any of them is removed they are no longer a generating set for $G$. There must be an element of maximal order, and assume after possibly permuting that it is $\phi(h_1)$. Then the cyclic group generated by $\phi(h_1)$ is a direct summand of $G$, thus we can take $\phi(h_1)$ as the first element of the basis for $G$ and pass to the quotient $G/\langle \phi_(h_1) \rangle$. Consider now the projection of $\phi(h_2), \ldots, \phi(h_m)$, and suppose $\phi(h_2)$ has maximal order in the quotient (thus again it is a direct summand in the quotient): then take $\phi(h_2)$ as second element element of the basis for $G$, and so on.

To my big surprise, I think this property need not be true when $G$ is not a $p$-group, although I am not sure I have a complete proof for this. My example is in the case $G=\mathbb{Z}_2 \oplus \mathbb{Z}_8$ $\oplus \mathbb{Z}_3 \oplus \mathbb{Z}27$ with the elements $\phi(h_1)= (1,2,0,1)$ and $\phi(h_2)=(0,1,1,3)$.

$\endgroup$
7
  • 2
    $\begingroup$ It is quite strange to answer your own question... $\endgroup$ Oct 3, 2012 at 13:19
  • $\begingroup$ You are right, but it seems nobody else cares. I have the feeling that everyone is convinced it is trivially true (which may very well be the case). $\endgroup$
    – calc
    Oct 3, 2012 at 13:24
  • $\begingroup$ It is a shame people can not vote for their own question and answer :) $\endgroup$
    – calc
    Oct 3, 2012 at 13:32
  • $\begingroup$ Can't you just use the Chinese Remainder Theorem to derive the general result from the result for $p$-groups? $\endgroup$
    – R.P.
    Oct 3, 2012 at 13:50
  • 5
    $\begingroup$ It is not strange to answer your own question: It is actually fairly common. $\endgroup$ Oct 17, 2012 at 14:23

Your Answer

By clicking “Post Your Answer”, you agree to our terms of service and acknowledge that you have read and understand our privacy policy and code of conduct.

Not the answer you're looking for? Browse other questions tagged or ask your own question.